Test Funda Nmat Mock-1

August 2, 2017 | Author: Abhishek Jain | Category: Zoo, Poaching, Sphere, Endangered Species, Geometry
Share Embed Donate


Short Description

Test Funda Nmat Mock-1...

Description

In a developing country, since 1960, the number of millionaires in an urban city has been increasing by 100% every 10 years while the number of millionaires of a rural town has been increasing by 200% every 10 years. In the year 2000, the sum of the number of millionaires in both the places was 337. What was the absolute difference between the number of millionaires in the city and the town in the year 1960? 1) 15 2) 16 3) 1 4) 9 5) 0 Solution: Let number of millionaires in the city and town in 1960 be x and y, respectively. An increase of 100% in x will make it 2x, while an increase of 200% in y will make it 3y, and so on. In 2000, the number of millionaires in the city = 2 × 2 × 2 × 2 × x = 16x And, the number of millionaires in the town = 3 × 3 × 3 × 3 × y = 81y ∴ Sum of millionaires in city and town = 16x + 81y = 337

The only set of integer values that satisfies this equation is x = 16, y = 1. ∴ Difference in number of millionaires in 1960 in the city and the town = 16 – 1 = 15 Hence, option 1.

2. 3 Marks

How many six-digit numbers are there such that the 3rd digit is the square of the 1st digit and the 4th digit is the square of the 2nd digit? 1) 256 2) 1000 3) 900 4) 6400 5) 1200 Solution: We need to find the number which is of the form aba2b2cd where a, b, c and d can take any value from 0 to 9. a and b cannot exceed 3 as the square of the two numbers then would exceed 9, which is not possible. ∴ a can have 3 values (1, 2 or 3) and b can have 4 values (0, 1, 2 or 3). c and d can have 10 values each. Thus total count of numbers that satisfy the given conditions is 3 × 4 × 10 × 10 = 1200

Hence, option 5.

In the above figure, AB is the diameter and CD is parallel to AB. Also BC = AD = 2. AB = 8. What is the length of CD? 1) 5 2) 6 3) 7 4) 4 5) None of these Solution:

3. 3 Marks

From the figure, O is the center of a circle and CE is perpendicular to OB. Let EB = x ∴ OE = 4 – x Applying Pythagoras theorem to ∆ OCE, we get, 42 = (4 – x)2 + CE2 ∴ CE2 = 8x – x2 Applying Pythagoras theorem to ∆ CEB, we get, 22 = x2 + 8x –x2 ∴ x = 1/2 But CD = AB – 2x = 8 – 1 = 7

Hence, option 3. Each question is followed by two statements, A and B. Answer each question using the following instructions: Mark (1) if the question can be answered by using statement A alone but not by using statement B alone. Mark (2) if the question can be answered by using statement B alone but not by using statement A alone. Mark (3) if the question can be answered by using either statement alone. Mark (4) if the question can be answered by using both the statements together but not by either of the statements alone. Mark (5) if the question cannot be answered on the basis of the two statements. If a and b are integers, what is the value of a + b? A. LCM of a and b is 34. B. One of the two numbers is 17. 1) 1 2) 2 3) 3 4) 4 5) 5 Solution: Using Statement A alone:

4. 3 Marks LCM(a, b) = 34

It is evident that the values of a and b are not unique. Thus, the question cannot be answered using statement A alone. Using Statement B alone: There is no information on the second number. Thus, the question cannot be answer using statement B alone. Using both the statements together: LCM(a, b) = 34 and one of these two numbers is 17. The other number could be 2 or 34. Hence, the sum a + b can still not be determined. Thus, the question cannot be answered on the basis of the two statements. Hence, option 5. 5.

Equation of a pair of line passing through the origin is 2x2 + xy − y2 = 0. What

3 are the separate equations of the lines? Marks 1) 2x + y = 0 and x − y = 0 2) 2x − y = 0 and x − y = 0 3) 2x + y = 0 and x + y = 0 4) 2x − y = 0 and x + y = 0 5) None of these Solution: 2x2 + xy − y2 = 0 is a second degree equation that represents a pair of straight lines passing through the origin. ∴ 2x2 + 2xy − xy − y2 = 0 2x(x + y) − y(x + y) = 0 ∴ 2x − y = 0 and x + y = 0 are the separate equations of the line. Hence, option 4.

1) 2 2) 3.6 3) 4.2 4) 5 5) 4 Solution: 6. 3 Mark s the line 4x + 3y = 14.

Hence, option 5. Each question is followed by two statements, A and B. Answer each question using the following instructions: Mark (1) if the question can be answered by using statement A alone but not by using statement B alone. 7. Mark (2) if the question can be answered by using statement B alone but not by 3 using statement A alone. Marks Mark (3) if the question can be answered by using either statement alone. Mark (4) if the question can be answered by using both the statements together but not by either of the statements alone. Mark (5) if the question cannot be answered on the basis of the two statements. Is 2x = y?

A. (2x − 9)2 = (y − 9)2 1) 1 2) 2 3) 3 4) 4 5) 5 Solution: Consider statement A alone: B.

Hence, (2x + y)2 = 8xy Thus, (2x − y)2 = 0 Or 2x = y Hence statement A alone is sufficient to answer the question. Consider Statement B alone: (2x − 9)2 = (y − 9)2 Thus, 2x − 9 = y − 9 or 2x − 9 = 9 − y Hence, 2x = y or 2x = 18 − y Hence, no definite conclusion is possible. Hence, we cannot answer the question using statement B alone. Hence, option 1. In Dexter‟s laboratory there is a specimen of bacteria that doubles its number every minute. On certain day, the number of bacteria at 6.10 pm was 1048576. What was the number of bacteria present at 6.00 pm the same evening? 1) 9216 2) 512 8. 3) 1024 3 4) 32 Marks 5) None of these Solution: Let the number of bacteria present at 6.00 pm be n. Thus at 6.10 pm it will be,

n(2)10 = 1048576 ∴ n = 1024 Hence, option 3. Group Question Answer the following questions based on the information given below. The following graph gives the sales revenue of five companies in the years from 2000 to 2004.

The maximum absolute decrease in the sales revenue of CD occurred between which of the following years? 1) 2000 and 2001 2) 2001 and 2002 3) 2002 and 2003 4) 2003 and 2004 5) 2001 and 2004 Solution: Note that the absolute decrease and not the percentage decrease is required.

9. 3 The sales revenue for CD has decreased only twice, i.e. between 2001 and Marks 2002 and between 2002 and 2003. Also, the sales revenue for CD in 2004 is more than the sales revenue in 2001. Hence, options 1, 3 and 4 can be eliminated.

From the year 2001 to 2002, absolute decrease registered for CD = 75 – 50 = 25 From 2002 to 2003, the absolute decrease = 50 – 0 = 50

∴ The absolute decrease in sales value of CD is maximum from 2002 to 2003. Hence, option 3. The absolute difference between the sales revenue of GH and IJ was the least in the year 1) 2000 2) 2001 3) 2002 4) 2003 5) 2004 Solution: The absolute difference between the sales revenue of GH and IJ for each year is given below. For 2000 = 100 – 50 = 50 10. 3 Marks For 2001 = 125 – 75 = 50 For 2002 = 100 – 100 = 0 For 2003 = 125 – 75 = 50 For 2004 = 100 – 75 = 25 ∴ The absolute difference between the sales revenue of GH and IJ was the least in the year 2002. Hence, option 3. In which of the following years was the ratio of the sales revenue of GH to that of EF the maximum? 1) 2000 2) 2001 3) 2002 4) 2003 5) 2004 Solution: The ratio of the sales revenue of GH to that of EF for each year is shown below.

11. 3 Marks For 2000 : 100/25 = 4

For 2001 : 125/50 = 2.5 For 2002 : 100/75 = 1.33 For 2003 : 75/50 = 1.5 For 2004 : 75/75 = 1

∴ The ratio of the sales revenue of GH to that of EF was maximum in the year 2000. Hence, option 1. Note: This question can be answered by direct observation as well. It is evident from the bar chart that the ratio is the highest in 2000. A boy, standing on top of a building 25 m high, notices a bird at some distance at an angle of elevation of 30°. At the same instant, he also notices reflection of the bird at an angle of depression of 45° in a small water pool, on the same level as the floor of the building and exactly vertically below the bird. What is the distance between the boy and the bird? 1) 2) 3) 4) 25 5) None of these Solution: Given data can be represented as shown in the following diagram.

12. 3 Marks

Let AB be the building, E be the location of the bird and C be the point on the pool on which the shadow of the bird is cast. The measure of all four angles of the quadrilateral ABCD is 90°, therefore, ABCD is a rectangle.

∴ AB = DC = 25 m In right angled ∆ADC, ∠DAC = 45° ∴ ∠ADC = 45° ∴ AD = DC = 25 m In ∆ADE, ∠DAE = 30°

Hence, option 2. The roots of the equation 4x4 − 4x3 − 72x2 − 4x + 4 = 0 are 1) 2) 3) 13. 3 Marks

4) 5) Solution: 4x4 – 4x3 – 72x2 – 4x + 4 = 0 Dividing by x2 on both the sides

∴ 4(a2 − 2) − 4(a) − 72 = 0 ∴ a2 − a − 20 = 0 ∴ a = 5 or a = −4 When a = 5

∴ x2 − 5x + 1 = 0

When a = −4

∴ x2 + 4x + 1 = 0

Hence, option 2. A piece of work when done by Amar, Akbar and Anthony respectively, costs Rs. 14. 540, Rs.324 and Rs. 320 respectively. The daily wages of Amar, Akbar and 3 Anthony are Rs.36, Rs.18 and Rs.32 respectively. What will be the total cost when MarksAmar, Akbar and Anthony are working together? 1) 480

2) 378 3) 387 4) 487 5) None of these Solution:

∴ The fraction of job that will be completed in a day when all three are working together is

∴ 4.5 days will be required to finish the job when all the three are working together.

Hence, option 3.

15. 3 Marks

Kishore owns a factory of plastic goods. His production cost comprises only the cost of raw material and labour . The cost of raw material increases by 20% and the labour cost also increases from 20% to 25% of the cost of raw material. By how much percent should he reduce his consumption of raw material and labour so that his production cost remains unchanged? 1) 20% 2) 15% 3) 17% 4) 25% 5) None of these Solution: Let the cost of raw materials be Rs. 100 Then the labour cost will be Rs. 20 ∴ Production cost = 120 Due the increase in cost of raw materials and labour cost the revised cost of raw material and labour cost will be Rs. 120 and Rs. 30 respectively. ∴ The new production cost will be Rs. 150 In order to bring back the production cost to Rs. 120 he should reduce the

consumption of raw materials and labour. ∴ Percentage reduction in consumption of raw material and labour will be,

Hence, option 1. What is the number of sides of a polygon for which sum of interior angles is 5400°? 1) 20 2) 32 3) 19 4) 18 5) None of these Solution: For an n sided polygon, there exist n vertices and therefore n interior angles. As we know, for every interior angle, there exists an exterior angle and thus n exterior angles exist.

16. 3 Marks Therefore, interior angle = 180 – exterior angle

It is given that the sum of the interior angles = 5400 ∴ 180(n – 2) = 5400 ∴ n = 32 Hence, option 2. Consider the following statements 17. 3 Mark s

Which of the statements is/are incorrect? 1) II only 2) III only 3) I only 4) I and II 5) II and III Solution:

Thus only statement II is incorrect. Hence, option 1. Jai and Veeru are two masons who are given the job of building a compound wall around a plot owned by Mr. Thakur. Jai working alone takes 8 hours more than the time that both Jai and Veeru would take working together. Veeru working alone takes 12.5 hours more than the time that both would take working together. How many hours would Jai alone take to build the wall? 1) 10 hours 2) 18 hours 3) 12 hours 4) 16 hours 5) None of these Solution: Let the time taken to build the compound when both Jai and Veeru are working together be x. 18. Time taken by Jai to complete the job alone is x + 8 and the time taken by Veeru 3 to complete the job alone is x + 12.5 Marks Thus we get the following equation:

∴ 2x2 + 20.5x = x2 + 20.5x + 100 ∴ x2 = 100 ∴ x = 10 Thus Jai will take 18 hours to complete the job alone.

Hence, option 2. Each question is followed by two statements, A and B. Answer each question using the following instructions: Mark (1) if the question can be answered by using statement A alone but not by using statement B alone. Mark (2) if the question can be answered by using statement B alone but not by using statement A alone. Mark (3) if the question can be answered by using either statement alone. Mark (4) if the question can be answered by using both the statements together but not by either of the statements alone. Mark (5) if the question cannot be answered on the basis of the two statements. Six friends Aziz, Vivek, Nipun, Arun, Sahil and Navnit are sitting around a circular table such that Arun is two places to the left of Aziz. Who is opposite Vivek? A. Sahil is adjacent to Nipun and Vivek B. Vivek is adjacent to Aziz and Sahil

19. 3 Marks

1) 1 2) 2 3) 3 4) 4 5) 5 Solution: The basic arrangement that can be obtained from the question is as shown below.

Using Statement A alone:

It is clear from the arrangement that either Arun or Aziz can be opposite Vivek. Thus, the question cannot be answered using statement A alone. Using Statement B alone: If Vivek is adjacent to Aziz and Sahil, Vivek has to be between Aziz and Sahil.

The only way this is possible is if Vivek is adjacent to Aziz on his right and Sahil is to the immediate right of Vivek. Now, Arun is opposite to Vivek as shown in the figure below.

Thus, the question can be answered using statement B alone. Thus, the question can be answered using statement B alone but not by using statement A alone. Hence, option 2.

20. 3 Marks

The difference between simple interest and compound interest on a particular amount in 2 years at 30% is Rs.360. What is the principal? 1) Rs. 4000 2) Rs. 3500 3) Rs. 3000 4) Rs. 2500 5) None of these Solution: The difference between the simple and compound interest for two years is the simple interest on the simple interest for one year.

∴ i = 1200 ∴ Interest for the first year = Rs. 1200

Hence, option 1. nth term of an AP, consisting of only positive integers, is denoted by an. It is given that a3 = 4 and aN – 2 = 9. What is the value of (aN + 2N + 2)? 21. 1) 29 3 2) 31 Marks 3) 33 4) 35

5) Cannot be determined Solution: Let common difference of the AP be d. ∴ We have a1 = a3 – 2d = 4 – 2d We also have a1 = aN – 2 – (N – 3)d = 9 – d(N – 3) = 9 – Nd + 3d Equating these two values of a1, we get, 4 – 2d = 9 – Nd + 3d, or (N – 5) d = 5 Since both N and d are positive, we must have N – 5 = 1 and d = 5 or N – 5 = 5 and d = 1. Thus, either N = 6 and d = 5 or N = 10 and d = 1. In the first case, aN + 2N + 2 = a1 + (N – 1) d + 2N + 2 = 4 – 2 × 5 + 5 × 5 + 2 × 6 + 2 = 33 In the second case, aN + 2N + 2 = a1 + (N – 1) d + 2N + 2 = 4 – 2 ×1 + 9 × 1 + 2 × 10 + 2 = 33 In either case, the given expression = 33 Hence, option 3. Each question is followed by two quantities, A and B. Answer each question using the following instructions: Mark (1) if quantity A is greater than quantity B. Mark (2) if quantity B is greater than quantity A. Mark (3) if the two quantities are equal. Mark (4) if it is impossible to determine a relationship. Mark (5) if the greater quantity cannot be determined but the two quantities are definitely not equal. In a new dictionary, the position of alphabets is changed so as to arrange the words more concisely. 22. 3 Let P be the old position of alphabets from 1 to 26, and let Pnew be the new Marks position. Pnew is calculated using the function f(P), defined as follows: f(P) = remainder of P × quotient of P, when divided by 3. 0. = quotient of P, when divided by 3.

... If none of them are

... If the remainder is

0. f(P) is calculated for all the values of P (i.e. 1 to 26) and arranged in the

ascending order in a set A. If f(P) for two values of P is the same, then the one with a higher quotient is placed after the one with lower quotient, and if they have the same quotient, then one with higher P is placed after the one with lower P. Now, the new position of a letter P is defined as the position of f(P) in the set A. A. Position of the word MAS in dictionary B. Position of the word MAN in dictionary 1) 1 2) 2 3) 3 4) 4 5) 5 Solution: Since the first two alphabets are the same, to find the order of the words, find the location of S and N in the new dictionary. P for S and N is 19 and 14 respectively. ∴ f(14) = 2 × 4 = 8 and f(19) = 1 × 6 = 6 ∴ f(19) < f(14) Hence, the word MAS is placed before the word MAN in the new dictionary. ∴ Position of the word MAN > Position of the word MAS Hence, option 2. Each question is followed by two statements, A and B. Answer each question using the following instructions: Mark (1) if the question can be answered by using statement A alone but not by using statement B alone. Mark (2) if the question can be answered by using statement B alone but not by using statement A alone. 23. Mark (3) if the question can be answered by using either statement alone. 3 Marks Mark (4) if the question can be answered by using both the statements together but not by either of the statements alone. Mark (5) if the question cannot be answered on the basis of the two statements. Is the 1st number in a series of 7 consecutive integers even? A. The arithmetic mean of the numbers is 2.

B. Sum of the numbers is divisible by 11. 1) 1 2) 2 3) 3 4) 4 5) 5 Solution: Using Statement A alone: Since the series consists of 7 consecutive integers, it forms an A.P. consisting of odd number of terms and the middle term of that A.P. is also the arithmetic mean of the A.P. Thus, the arithmetic mean of the 7 numbers is the 4th number which is 2. Therefore, the 1st number in the series is 2 − 3 = −1. Thus, the 1st number is odd. Thus, the question can be answered using statement A alone. Using Statement B alone: Sum of the 7 numbers = (x – 3) + (x – 2) +( x – 1) + x + (x + 1) + (x +2) + (x + 3) = 7x Thus, the sum of the given numbers is divisible by 7. Now, it is known this sum is also divisible by 11 which means that x has to be a multiple of 11. But x can be odd or even which implies that the first term can also be odd or even. Thus, the question cannot be answered using statement B alone. Thus, the question can be answered using statement A alone but not by using statement B alone. Hence, option 1. A train moving at 54 kmph crosses a man standing on a platform in 10 seconds. What will be the time required by the train to cross a platform if the length of the 24. platform is equal to the length of the train? 1) 30 seconds 3 Marks 2) 50 seconds 3) 25 seconds 4) 20 seconds 5) 45 seconds

Solution:

Distance covered by the train to cross a man standing on the platform = Length of the train ∵ Distance = Speed × Time ∴ Distance covered = 15 × 10 = 150 m So the length of the train is 150 m. Length of the platform = Length of the train = 150 m Time taken by the train to cross the platform = Total Distance covered (Length of platform + Length of train)/Speed of train

Hence, option 4.

Alternatively, As the length of the platform is equal to that of the train, the train will require twice the time to cross it as it takes to cross the man. Hence, option 4. There are 3 friends competing against each other in a race. Mohit beats Purohit in a race of 1000 m by 50 m and Purohit beats Rohit in a race of 1200 m by 100 m. By what margin (in m)would Mohit beat Rohit in a race of 1500 m? (Round off the answer to closest integer.) 1) 190 2) 194 3) 197 25. 4) 198 3 5) 199 Marks Solution: Let the speed of Mohit, Purohit and Rohit be m, p and r respectively. In a race of 1000 m,

In a race of 1200 m,

Let Mohit beats Rohit by x in a race of 1500 m .

∴ 5225 = 6000 – 4x ∴ 4x = 775 or x = 193.75 Hence, option 2. What is the ratio of the right most digit preceding the zeros in the value of 2053 to the right most digit preceding the zeros in the value of 4053. 1) 1 : 2 2) 2 : 3 3) 2 : 1 4) 3 : 2 5) None of these Solution: 2053 = 253 × 1053 26. 3 ∴ The rightmost digit preceding zeros will be the unit's digit of 253 Marks 21 = 2, 22 = 4, 23 = 8, 24 = 16, 25 = 32 and so on. Thus we can see that the units digit repeats after every 4th power. 53 = 52 + 1, where 52 is a multiple of 4. ∴ The unit's digit of 253 is same as the unit's digit of 21, which is 2. Similarly we can show that the right most digit preceding the zeros in the value

of 4053 is 4. Hence, the ratio is 1 : 2 Hence, option 1. In the figure below, m∠ABC = 40°, m∠BAD = 140°, m∠ADE = 130° and DC = DE. Find m∠CDE + m∠DCB.

1) 160° 2) 170° 3) 210° 4) 150° 5) None of these Solution: In quadrilateral ABED,

27. 3 Marks m∠ABE + m∠BAD + m∠ADE + m∠DEB = 360° ∴ m∠DEC = 50° Since DC = DE ∴ m∠DEC = m∠DCE = 50° Now, in ∆DCE, m∠DCE + m∠DEC + m∠CDE = 180° ∴ m∠CDE = 80° Now, m∠DCE + m∠DCB = 180° ∴ m∠DCB = 130° ∴ m∠CDE + m∠DCB = 210° Hence, option 3. 28.

In class A, the ratio of the average age of boys to the average age of girls is 1 : 2

3 and the ratio of number of boys to the number of girls is 2 : 1. In class B, the ratio Marks of average age of boys to the average age of girls is 3 : 2 and the ratio of number of boys to the number of girls is 1 : 2. The number of students in class A and class B are equal. If the average age of class A is equal to the average age of class B, then what is the ratio of the average age of girls of class A to the average age of boys of class B? 1) 7 : 6 2) 6 : 5 3) 5 : 3 4) 2 : 3 5) 5 : 4 Solution: For class A: Let the average age of boys be x and the number of girls be y. ∴ Average age of girls is 2x and the number of boys is 2y.

For class B: Let „z’ be the common variable for average age of boys and girls. ∴ Average age of boys is 3z and average age of girls is 2z. Since, the total number of students is equal in both classes, the number of boys is y and the number of girls is 2y.

Now, since the average ages of both classes are equal

Ratio of average age of girls of class A to the average age of boys of class B is

Hence, option 1. 29. 3

In Vrindavan Housing Society the monthly society expenditure has a fixed and a

Marks variable component, such that the variable component depends on the number of members in the society. If there are 50 members, then each member has to pay Rs. 110 and if there are 30 more members then each member has to pay Rs. 80. If each member pays Rs. 130, then how many members are there in the society? 1) 45 2) 55 3) 40 4) 60 5) None of these Solution: It is given that, expenditure = F + V From the condition given we can say that, (110)(50) = F + 50k

...(i)

(80)(80) = F + 80k

...(ii)

On solving (i) and (ii) we get, F = 4000 and k = 30 Now, 130x = 4000 + 30x, where x is the number of members ∴ x = 40 Hence, option 3. Group Question Answer the following questions based on the information given below. The graph below gives details of total exports as well as exports of herbal products for a company. The abscissa represents the total exports for a given year in Rs. Lakhs, while the ordinate represents the exports of herbal products for that same year in Rs. Lakhs.

For 2006, herbal products formed what percentage of the total exports? 1) 13.50% 2) 13.64% 3) 14.28% 4) 12.50% 5) 11.11% Solution: The abscissa and ordinate stand for the X-axis and Y-axis respectively. 30. Thus, figures on the X-axis stand for the total exports for a given year while 3 Marks those on the Y-axis stand for the export of herbal products in that year. Thus, in 2006, the total exports were Rs. 5,500 lakhs, whereas the exports of herbal products were Rs. 750 lakhs.

Hence, option 2. The fall in herbal exports in 2007 from that in 2006 was nearly: 1) 13.65% 2) 12.78% 3) 13.33% 4) 86.66% 5) 90.25% 31. Solution: 3 Marks In 2006, the herbal exports were Rs. 750 lakhs, whereas in 2007, they were Rs. 650 lakhs.

Hence, option 3. Over the given period, the herbal exports rose by nearly: 1) 300% 2) 30% 3) 10% 4) 18.65% 5) 77% 32. Solution: 3 In 2004, the herbal exports were Rs. 500 lakhs while at the end of the period, i.e. in Mark 2007, they were Rs. 650 lakhs. s

Hence, option 2. In how many ways can a committee of 4 people comprising at least 3 boys be formed using a group of 5 boys and 6 girls? 1) 65 2) 34 3) 60 4) 46 5) None of these Solution: Atleast 3 boys means the committee can be formed by 3 boys and 1 girl or all 4 boys. 33. 3 Case 1: 3 boys and 1 girl Marks From 5 boys and 6 girls they can be selected in 5C3 × 6C1 ways = 10 × 6 = 60 ways Case 2: All 4 boys They can be selected from 5 boys in 5C4 ways = 5 ways ∴ Total number of ways = 60 + 5 = 65 ways Hence, option 1. A, B and C invest Rs. 4000, Rs. 15000 and Rs. 5000 respectively to set up a new business. A gets 20% of the total profit for running the business and the remaining profit is divided among the three in proportion to their investments. If in a year, A 34. gets Rs. 3000 less than B and C together, the total profit for the year is 1) Rs. 5000 3 Marks 2) Rs. 6000 3) Rs. 7500 4) Rs. 9000 5) Rs. 10500 Solution:

Ratio of investments of A, B and C ≡ 4000 : 15000 : 5000 ≡ 4 : 15 : 5 Let the profit for the year be Rs. x Profit given to A for managing the business = 20% of x = 0.2x 80% of the total profit is divided between A, B and C in ratio 4 : 15 : 5 80% of total profit = 0.8x

A earns Rs. 3000 less than B and C.

∴ x = 9000 Hence option 4. In ∆PQR, a line is drawn from P to intersect the opposite side QR at S. What is m ∠QPS such that PR = RS and m ∠RPQ = m ∠PQR + 60°? 1) 15° 2) 30° 3) 20° 4) 25° 5) 45° Solution: 35. 3 Marks

m ∠QPS = m ∠RPQ − m ∠RPS

∵ PR = RS ∴ m ∠QPS = m ∠RPQ – m ∠PSR Using exterior angle theorem, we get, m ∠PSR = m ∠QPS + m ∠PQS ∴ m ∠QPS = m ∠RPQ – (m ∠QPS + m ∠PQS) ∵ m ∠PQS = m ∠PQR ∴ 2(m ∠QPS) = m ∠RPQ − m ∠PQR ∴ 2(m ∠QPS) = 60° ∴ m ∠QPS = 30° Hence, option 2. A field is in the form of a circle of radius 100 m; the field is covered with grass except for an area in the form of a circle concentric with the field, and of radius 50 m. A parachutist is falling towards the field because his parachute has failed to open; he knows that he will die unless he falls on grass. He may fall anywhere on the field. What is the probability that he will live? 1) 2) 3) 4) 36. 5) 1 3 Solution: Marks The probability that the parachutist will fall at any point on the field is equal to the probability that he will fall at any other point on the field. Thus, the probability that he will fall on grass (and hence live) is equal to the area of the grassy portion of the field divided by the total area of the field. The area (in sq. m.) of the grassy portion = π[1002 – 502] The area (in sq. m.) of the entire field = π[1002]

Hence, option 3. Gayathri and Savithri sell apples. Savithri sells two apples for one rupee. The apples that Gayathri sells are a bit smaller; she sells three apples for one rupee. At a certain moment, when both ladies have the same amount of apples left, Gayathri is called away. She asks her neighbour to take care of her goods. To make everything not too complicated, Savithri simply puts all apples together, and starts selling five apples for Rs. 2. When Gayathri returns the next day, all apples have been sold. But when they start dividing the money, there appears to be a shortage of Rs. 7. Supposing they divide the amount equally, how much does Savithri lose in this deal? 1) 12 2) 21 3) 0 4) 25 5) Cannot be determined Solution: The big pile of apples contains the same number of large apples of half a rupee each (from Savithri), as smaller apples of one third a rupee each (from Gayathri).

37. 3 Marks ∴ The apples should be sold at Rs. (5/12) each. But the apples were sold for Rs. (2/5) each (5 apples for Rs. 2). ∴ Each apple is sold at Rs. (1/60) less. The total shortage is Rs. 7, ∴ After combining there were 7 × 60 = 420 apples.

∵ They divided the money equally amongst themselves, each of them got Rs. 84. If Savithri would have sold the apples herself, she would have received Rs. 105 for 210 apples. ∴ Savithri loses Rs. 21 in this deal. Hence, option 2. A number which reads the same when read forward and backward is called a 38. palindrome. 3 How many four-digit numbers are palindromes? Marks 1) 81 2) 90

3) 99 4) 100 5) 115 Solution: A 4-digit palindrome will be of the form abba where a > 0 and b can take integer values from 0 to 9. ∴ a can take 9 values and b can take 10 values. ∴ The total number of 4-digit palindromes is 9 × 10 = 90 Hence, option 2. Seven lotuses, sixteen roses and nine lilies cost Rs. 338. Four lotuses, six roses and seven lilies cost Rs. 169. What is the cost of nine lotuses, thirty roses and six lilies? 1) Rs. 169 2) Rs. 507 3) Rs. 496 4) Rs. 570 5) Cannot be determined Solution: Let the price of a lotus, a rose and a lily be Rs. x, y and z respectively. 39. ∴ 7x + 16y + 9z = 338 3 Marks and 4x + 6y + 7z = 169

...(i)

...(ii)

Subtracting (ii) from (i) and multiplying the resulting equation by 3, we get 9x + 30y + 6z = 507 So the cost of 9 lotuses, 30 roses and 6 lilies is Rs. 507. Hence, option 2. In a survey conducted 70% watched CNN, 75% watched BBC, 20% watched neither of the channels and 325 people watched both the channels. How many people were surveyed? 40. 1) 500 3 2) 382 Marks 3) 50 4) 492 5) 491 Solution:

Let x be the total number of people surveyed. From the figure above, 0.7x – 325 + 325 + 0.75x – 325 + 0.2x = x ∴ x = 500 Hence, option 1. The length of the side of a cubical box is equal to twice the length of the longest rod that can be placed in a rectangular box of dimensions 3 cm × 4 cm × 12 cm. What is the maximum number of spheres of diameter 32.5 mm that can be kept in the cubical box? 1) 64 2) 512 3) 216 4) 1000 5) 324 Solution: The length of the longest rod that can fit in the cuboid 41. = The diagonal of the cuboid 3 Marks

Side of the cubical box = 2 × 13 = 26 cm

∴ Along one face of the cube 8 × 8 sheres can be fitted as shown in the figure given below.

Also, there will be 8 such vertical slabs in the cube. ∴ The total number of spheres that can be fitted in the cube = 8 × 8 × 8 = 512 Hence, option 2. If m and n are two natural numbers, ratio of m to n is x and ratio of n to m is y, then (x + y) will be 1) Less than 1 2) Greater than 1 3) Equal to 1 4) Depends on m and n 5) Data insufficient Solution: 42. 3 Marks ∴ x + y > 1

Even then x + y > 1 ∴ x + y is always greater than 1. Hence, option 2.

There are three identical circles of radii 10 units touching each other. What is circumference of the biggest circle that can fit in the gap formed by the intersection of all three circles and touching all three of them? 1) 2) 3) 4) 5) None of these Solution:

43. 3 Marks Let the radius of a smaller circle be r. ∆ABC is an equilateral triangle.

∵ the centroid divides the median of a triangle in the ratio 2 : 1,

∴ Circumference = 2πr

Hence, option 2. On a busy Monday morning a certain number of cyclists started from Borivali towards Churchgate. At the Malad junction, half of these cyclists entered Malad while 1/5 of the remaining cyclists joined the cyclists going towards Chuchgate. The same thing happened at Andheri, Santacruz, Bandra and Dadar. No cyclist joined or left the group thereafter. Finally 243 cyclists reached Churchgate. How many cyclists started from Borivali? 1) 3125 2) 1875 3) 2225 4) 2875 5) None of these Solution: There are 243 cyclists at Churchgate. So 243 cyclists left Dadar. Let there be x cyclists before Dadar. 44. Since half of the cyclists left, there were x/2 cyclists remaining. 3 Marks These were joined by 1/5 of remaining cyclists.

This has to be equal to 243. ∴ x = 405 Using the same logic, we have the number of cyclists before Bandra to be 675, before Santacruz to be 1125, before Andheri to be 1875 and before Malad to be 3125. Thus total cyclists leaving Borivali is 3125. Hence, option 1. Ram invested Rs. 10,000 each in two bank schemes. In scheme A he earns an interest of 10% p.a., compounded semi-annually, while in scheme B he gets a simple interest of 6% every six months. What is the difference in the interest that 45. he will earn at the end of 2 years? 3 1) 245 Marks 2) 200 3) 260 4) 243 5) 267

Solution: Scheme A: The interest is compounded semi-annually. Interest earned at the end of 2 years

= 10000 × 1.054 – 10000 ≈ 10000 × 1.2155 – 10000 = 2155 Scheme B: The rate of interest is 6% per 6 months i.e. 12% p.a. ∴ The interest earned in 2 years = 0.12 × 2 × 10000 = 2400 ∴ Difference between the interest earned = 2400 – 2155 = Rs. 245 Hence, option 1. Group Question Answer the following questions based on the information given below. In the year 2004, Deepesh and Devesh together purchased 20 acres of land from Dinesh. They paid amounts in the ratio 4 : 1. Further Devesh spent Rs. 3 lakhs more on the land for its development, and thereby they spent equal amounts. They cultivated mango and orange trees in equal areas. However the ratio of number of mango trees to that of orange trees was 3 : 2. In the year 2008, the trees yielded fruits. They sold each mango for Rs. 10. The revenue obtained from the total sale of mangoes and oranges was 25% of the purchase value of the land. They agreed to share the realised value equally. The total amounts generated from the sale of mangoes and oranges were in the ratio of 3 : 2. What is the ratio of revenue per acre of oranges to that of mangoes? 1) 3 : 2 2) 2 : 3 46. 3) 1 : 1 3 4) 4 : 9 Marks 5) 9 : 4 Solution: ∵ Oranges and mangoes are cultivated on equal areas, the ratio of revenue

per acre of oranges to that of mangoes is the same as the ratio of revenue from oranges to that of mangoes, which is 2 : 3. Hence, option 2. The amount received (in Rs.) by Devesh in the year 2008 is 1) 62500 2) 750000 3) 100000 4) 125000 5) 300000 Solution: Let Deepesh pay 4x lakhs and let Devesh pay x lakhs for the land. Devesh spends Rs. 3 lakhs more.

47. 3 ∴ 4x = x + 3 i.e. x = 1 Marks

Total purchase value of the land = 4x + x = 5 lakhs Total amount realized from sale of both mangoes and oranges = 25% of 5 lakhs = Rs. 1.25 lakhs Amount received by Devesh in the year 2008 = 125000/2 = Rs. 62,500 Hence, option 1. The total number of mangoes produced is 1) 5000 2) 50000 3) 6250 4) 62500 5) 7500 Solution: From the solution to the previous question we get that the total revenue generated from the sale of mangoes and oranges = Rs. 1.25 lakhs 48. 3 Revenue from mangoes = (3/5) × 1.25 lakhs = Rs. 75,000 Marks Revenue from oranges = (2/5) × 1.25 lakhs = Rs. 50,000 Total revenue from mangoes = Rs. 75,000 Selling price of each mango = Rs. 10 Number of mangoes = 75000/10 = 7500 Hence, option 5.

Section II 1. Answer the following questions based on the information given below. 3

Marks With or without religion, you would have good people doing good things and evil people doing evil things. But for good people to do evil things, that takes religion. The above argument rests on which of the following assumptions?

1) Religious people do evil things. 2) Good people are incapable of doing evil things. 3) Every evil deed has religion behind it. 4) People are either good or evil. 5) Good and evil can be measured quantitatively. Solution: It is not assumed that religious people generally do evil things, as stated in option 1. The given information only states that for good people to do evil, it requires religion - this cannot be generalized to ALL religious people. Option 3 is not assumed because evil people without religion can also commit evil deeds. Option 4 is not an assumption because although the argument is about good and evil people, it does not assume that these are the only two classes in which people can fall. Option 5 introduces „quantitative‟- no relation to maindata. Option 2 is an assumption because the paragraph states if good people have to do evil things, they are forced by religion to do so- hence they are generally incapable of doing evil (incapable means: lacking capacity, ability, or qualification for the purpose or end in view). Hence, the correct answer is option 2. During the Physical Training session in a school, some girls were made to stand in a straight line. Asha was the 4th girl from the front, while Nisha was the 3rd last girl. The instructor then asked 5 consecutive girls from the center of the line to move out of the line. This left exactly 4 girls between Asha and Nisha. How many girls were in the line initially? 1) 14 2) 15 3) 16 4) 17 5) 18 Solution: 2. Asha was the 4th girl from the front which means that there were 3 girls in front 3 Marks of Asha. Nisha was the 3rd last girl which means that there were 2 girls behind Nisha. Thus, the number of girls who are always present in the line are 3 + 1 + 1 + 2 i.e. 7. After the instructor asked 5 consecutive girls to move out, there were 4 girls left between Asha and Nisha. i.e. initially there were 9 girls between Asha and Nisha.

Thus, the number of girls initially in the line = 7 + 9 = 16. Hence, option 3. Given below is a passage followed by several statements that can be drawn from the facts stated in the passage. Examine each statement separately in the context of the passage and decide whether they are implied from the passage. First, let us do away with the myth that parents teach language to their children. No one supposes that parents provide explicit grammar lessons, of course, but many parents (and some child psychologists who should know better) think that mothers provide children with implicit lessons. These lessons take the form of a special speech variety called Motherese: intensive sessions of conversational give-and-take, with repetitive drills and simplified grammar. („Look at the doggie! See the doggie? There‟s a doggie!‟) Statements:

3. 3 Marks

I. II. III.

Some parents mistakenly think that they teach their children language. Motherese may not particularly helpful. Child psychologists are expected to be aware that parents provide children with only implicit language lessons.

1) Only I is implicit. 2) Only III is implicit. 3) Both I and II are Implicit. 4) Both II and III are Implicit. 5) All are implicit. Solution: I is very much implicit in the first sentence, and forms the basis for the whole argument. Since the idea of parents teaching their children language is called a „myth‟, we can infer that even a special variety of speech like Motherese is not particularly helpful in doing so. So II is implicit. III directly contradicts the parenthetical observation in the second sentence, so it cannot be said to be implicit. Therefore only assumptions I and II are implicit. Hence, the correct answer is option 3. Group Question Answer the following questions based on the information given below. In Ramraj society, it is known that only three brands of vehicles are used, i.e. Hero Honda, Yamaha and Bajaj. 42 families use only one brand, 23 families use exactly two brands and 10 families use all three brands. It is assumed that each family uses at least one of these brands. If 8 families stop using Yamaha and start using Bajaj, then what can be the 4. maximum number of families that use exactly two brands? 3 1) 26 Marks 2) 32 3) 30

4) 31 5) 23 Solution:

Since every family uses atleast one brand, there is no family that does not use any brand. 42 families use only one brand ∴ a + b + c = 42 23 families use exactly two brands ∴ d + f + e = 23 10 families use all the three brands. ∴ g = 10 The 8 families that stop using the Yamaha brand and start using the Bajaj brand can be from any one of the three categories : single brand users, two brand users or all brand users. Since the maximum number of families that use two brands is required, this can happen only if all the 8 families are from the same category. Therefore, there are three cases possible. Case 1 : All 8 families are single brand users. In this case, a + b + c is still 42 because the 8 families get reduced from „Yamaha only‟ and get added to „Bajaj only‟. Thus, there is no impact on the number of families using two brands i.e. its value still remains 23. Case 2 : All 8 families are two brand users. In this case, d + e + f can have a maximum value of 23 because in a best case scenario, there are originally no families in the „Yamaha and Bajaj only‟

category and the 8 families get reduced from the „Hero Honda and Yamaha only‟and get added to „Bajaj and Hero Honda only‟ category. If there were some families in the „Yamaha and Bajaj only‟ category, then after the shift, the number of families in this category would have become zero while the number of families in the „Bajaj only‟ category would have increase by 8. Thus, the number of families that use two brands would have reduced by 8 i.e. become 15. Thus, in this case, the value becomes either 15 or remains 23. Case 3 : All 8 families use all three brands. Here, the value of g definitely decreases by 8 to become 2. This is because the families in this category now become two brand users i.e. they fall in the category „Bajaj and Hero Honda only‟. Hence, the number of families using two brands becomes 23 + 8 = 31. Hence, the maximum value is 31. Hence, option 4. How many families do not use all the three brands? 1) 50 2) 60 3) 54 4) 65 5) None of the above 5. Solution: Since every family uses at least one brand, the total number of families that 3 Marks uses any one of the brands is 42 + 23 + 10 = 75. The number of families that uses all three brands = 10 ∴ Number of families that don‟t use all three brands = 75 – 10 = 65 Hence, option 4.

6. 3 Marks

Answer the question based on the information given in the passage. One reason given by those who are pro-zoos is that zoos have a very important role to play in breeding endangered animals in captivity. According to pro-zoo people, if animals were being taken care of in their natural environment, i.e. in forests and oceans, then there would have been no need for zoos. But so many animals are now becoming extinct and it is up to the zoos to somehow salvage the situation. Which of the following statements further weakens the argument presented in the passage? 1) Zoos are primarily seen as entertainment centres. Zoos do not offer guided tours to visitors, therefore there has been no rise 2) in awareness.

Studies have found that city zoos play an important role in protecting endangered species from poachers. 4) Zoos have been able to save many species from the brink of extinction. Studies show that zoos have a very low success rate when it comes to 5) conserving endangered animals, due to diseases and defects caused due to inter-breeding. Solution: The main idea expressed in the passage is that zoos play a very important role in conserving endangered species. To weaken this argument we need a statement that shows that zoos are not doing this, or are somehow incapable of doing this. Option 1 is ruled out as it is not relevant to the argument in the passage about conserving endangered species. Option 2 is ruled out as there is no relation between awareness and wildlife conservation in this passage. Option 3 is ruled out as this is just a rephrasing of what has been stated in the passage; it doesn‟t weaken the argument. Option 4 is ruled out as this would not weaken the argument, rather it would strengthen it. If the zoos have a very low success rate in conserving animals and inter-breeding is leading to diseases and defects, then zoos are not playing a role in conserving endangered species. This certainly weakens the given argument. Hence, the correct answer is option 5. 3)

Given below are two statements. Analyze them and mark the option that correctly states their relationship. A. I wrote pages after pages on the experiences of my travels. B. The ink-pot fell from the table and the ink in the pen ran dry. 1) A is the cause and B is the effect. 2) B is the cause and A is the effect. 3) Both A and B are effects of a common cause. 7. 4) Both A and B are the causes of a common effect. 3 Marks 5) Cannot be determined. Solution: This is an abstract set of statements that connect logically only on the basis of certain assumptions. On the face of it, statement A looks like the cause to the fact mentioned in statement B. However, we have to assume that the author was using to write the very pen that ran out of ink and he did something for the ink-pot to fall. These two statements cannot be assumed to be related to each other as it would render the relationship ambiguous. Hence, the correct answer is option 5.

8. 3 Marks

Read the two statements given below and choose the statement that logically follows from the first two. A. All pygmies are brown. B. All pygmies are black. 1) 2) 3) 4)

Brown is same as black. Some brown is black. Some black are not brown. Some brown are pygmies.

5) Every brown is black. Solution:

The two given statements can be represented in the form of a Venn diagram. Option 1 is incorrect because brown and black may not be the same set. Option 2 is correct because there has to be some brown that are common to black (the set of all pygmies). Option 3 is incorrect because brown and black could be the same set. Option 4 is derived only from statement A and is hence, incorrect. Option 5 is incorrect as seen in the Venn Diagram. Hence, option 2. The statement given below is followed by two statements. Analyze all of them and determine the correct combination of a course of action. Illegal hunting of tigers has caused the tiger population of India to reach alarming levels. 9. 3 Marks

A. Environmental groups should spread awareness about this fact. B. The government should enforce stricter measures towards upholding the existing tiger poaching laws. 1) 2) 3) 4) 5)

A is the best course of action. B is the best course of action. A followed by B is the best course of action. Either A or B is the best course of action. Neither A nor B are relevant courses of action.

Solution: Both of these statements seem useful as courses of action when it comes to tiger poaching. However, this situation is very specific and so generic actions are not relevant in this case. Statement A will definitely result in people being more aware of dropping tiger populations and start caring more but such awareness among the general population may or may not actually reduce the number of tigers being hunted illegally. Statement B, on the other hand, targets the crux of the problem, that is upholding laws more strictly so that the people who hunt these tigers are caught and stopped from hunting any more tigers. Between these two, statement B is more relevant as a course of action than statement A. Hence, the correct answer is option 2. In each of the following questions, there is a statement followed by a set of arguments. You are expected to classify the argument as Strong or Weak. Strong arguments are important and directly related to the question. Weak arguments may not be directly related or may be related to trivial aspects of the question and may be of lesser importance. Choose the best option as per this classification. Should there be a statutory limit on the amount spent on weddings, in view of the display of wealth witnessed at the weddings of politicians and industrialists? I.

II. 10. 3 Marks

Yes. Conspicuous consumption by public authorities undermines efforts to cultivate healthy economic habits across all classes, which is important for the success of the nation. No. A large wedding creates jobs for people in several industries, from florists to caterers to photographers to bridal fairs. It boosts the local economy.

1) Only I is strong. 2) Only II is strong. 3) Both I and II are strong. 4) Either I or II is strong. 5) Neither I nor II is strong. Solution: Argument I is not supported by strong facts. Firstly, in a democracy, one cannot lay a limit on the personal spending behaviour of a person. Secondly, across all classes is incorrect. People from the middle and lower economic classes will not have lavish spending habits, therefore, do not need to take an effort to cultivate healthy economic habits with respect to lavish weddings. Hence, argument I is weak. Argument II is supported with unimpeachable facts. Therefore, argument II is strong. Hence, the correct answer is option 2. Group Question Answer the following questions based on the information given below. A clerk has been given the task of arranging a round table conference that will be attended by 8 delegates from different countries : A, B, C, D, E, F, G and H.

The main job of the clerk is to assign the name of the delegate to the chair meant for that delegate. The clerk has been given the following information – i. ii. iii. iv. v. vi.

The chairs are numbered from 1 to 8. The delegate from country A will sit opposite to the delegate from country E. The delegate from country C sits adjacent to the delegate from country D. The delegate from country F sits opposite to the delegate from country G. The delegate from country B sits on the right hand side of the delegate from country A. The delegate from country H sits adjacent to the delegate from country E.

The clerk has been further informed that the delegate from country A sits on the seat numbered 1 and that the seats have been arranged in clockwise fashion. Try helping the clerk in this job by answering the following. Whose name will the clerk write on seat number 2? 1) Delegate from G 2) Delegate from F 3) Delegate from D 4) Delegate from C 5) Cannot be determined Solution: Since it is given that the delegate from country A sits on seat number 1 and that the seats are numbered in clockwise order, fix A's position and number the other seats accordingly. Now, the delegate from country E sits exactly opposite to the delegate from country A. Thus, the delegate from country E sits on seat number 5. 11. 3 Since the delegate from country B sits on the right hand side of the delegate Marks from A, the delegate from B can sit on any one seat from seat numbers 6, 7 and 8. Now, the delegates from countries C and D sit adjacent to each other and the delegates from countries G and F are exactly opposite each other. This implies that the delegates from countries C, D and one out of countries G and F have to occupy three adjacent seats. Hence, the three delegates mentioned above have to occupy seat numbers 2, 3 and 4 (in no particular order). Now, the delegate from country H sits adjacent to the delegate from country E.

This means that the delegate from country H can sit on one of seats 4 and 6. But, as seen above, seat 4 will be occupied by one delegate from C, D and G/F. Hence, the delegate from country H will be on seat number 6. Consequently, the delegate from country G or F cannot be opposite to seat number 6 i.e. on seat number 2. Similarly, this delegate cannot be on seat number 3 because the delegates from C and D have to be together. Hence, one delegate from country G and F will be on seat number 4 while the other will be opposite him i.e. on seat number 8. Consequently, the delegates from countries C and D will be on seat numbers 2 and 3 (in no particular order). Hence, the delegate from country B can only be in seat number 7. Thus, the final arrangement is as shown below.

Therefore, it is clear from the figure that seat number 2 will be occupied by either C or D. Hence, option 5. Who will sit on the seat number 7? 1) Delegate from G 2) Delegate from H 12. 3) Delegate from F 3 Marks 4) Delegate from B 5) Cannot be determined Solution: Consider the final arrangement obtained in the solution to the first question.

It is evident that the delegate from country B sits on the seat numbered 7. Hence, option 4. On which seat number will H be seated? 1) 4 2) 6 3) 7 4) 2 13. 5) Cannot be determined 3 MarksSolution: Consider the final arrangement obtained in the solution to the first question. You can observe that H sits on seat number 6. Hence, option 2. Read the two statements given below and choose the statement that logically follows from the first two. A. All Foreigners are Stupid. B. All Stupid are Awkward. 1) Some Awkward are Stupid. 2) Some Foreigners may be Stupid but not Awkward. 3) Some Stupid are Foreigners. 4) All Awkward are Foreigners. 5) Some Awkward are Foreigners. Solution:

14. 3 Marks

The two given statements can be represented in the form of a Venn diagram as shown.

Options 1 and 3 are incorrect as they are derived from one of the given statements only. Option 2 is incorrect as such a case is not possible (as seen in the Venn Diagram). Option 4 is not correct. Though it may be one of the possible cases, it is not necessarily true. Option 5 is correct as can be seen from the diagram. Hence, option 5. A coding machine generates a code for words in a certain manner such that the code for „STRANGER‟ is 19-T-18-A-14-G-5-R. What will be the code generated by this machine for the word „IMPORTANT‟? 1) 10-M-16-O-18-T-1-N-20 2) 9-M-16-O-18-T-1-N-21 3) 9-M-14-O-18-T-1-N-20 4) 9-M-16-O-18-T-1-N-20 5) 4-I-1-L-16-G-21-E Solution: Looking at the code for „STRANGER‟, it is clear that starting from the first letter, every alternate letter is replaced by its position in the alphabet. The other set of alternate letters stay as they are. Word: 15. 3 Position: Marks Code:

S

T

R

A

N

19 na 18 na 14 19

T 18

A

14

G

E

R

na 5

na

G

5

R

A

N

Now, consider the word „IMPORTANT‟. Word: T

I

M

P

Position:

9

na 16

na 18

Code:

9

M

O 18

16

O

R

T

na 1 T

1

na N

20 20

∴ The code for „IMPORTANT‟ is 9-M-16-O-18-T-1-N-20. Hence, option 4. Read each of the passages and answer the questions that follow. 16. Today‟s society is reeling from the effects of constant advertising messages. 3 These advertisements promote harmful and dangerous things that weaken social Marks fabric and lead to the ill-health of young people. Advertisers promote unhealthy things and are not genuinely concerned about society. Fast food adverts are a

major reason why children today are overweight. Which of the following statements, if true, best weakens the argument presented in the passage? 1) Advertising affecting obesity is just a myth. 2) Society cannot deal with the pressure of advertising. 3) Fast food is the leading cause of obesity. Researchers have found that most young people tend to be quite lean till 4) the age of 20. Advertisements that promote unhealthy products are not as common as 5) believed and fast food companies are now promoting their healthier dishes on offer. Solution: The main idea presented in the passage is that advertisements promote dangerous things that have a negative impact on people. The example is that of fast food advertisements targeted at young people. To weaken the argument we need a statement that shows that all advertisements do not promote unhealthy things and that fast food companies do not promote fattening food products which leads to children being overweight. Option 1 does not weaken the argument as there is no concrete data. Option 2 simply rephrases what has already been stated in the passage. Option 3 supports the passage. Option 4 is unrelated to the effects of advertising and merely adds more data to the passage. Option 5 is the correct answer since it weakens the argument by stating that advertisements that promote unhealthy products are rare (this goes against the statement that, „Today‟s society is reeling from the effects of constant advertising messages‟). Secondly, it also says that fast-food companies have been promoting their healthier dishes, so the advertisements promoting these kinds of food cannot be held accountable for obesity among the young. Hence, the correct answer is option 5. Group Question Answer the following questions based on the information given below. Ten friends go to watch the IPL match. The seating arrangement is such that 5 people sit in one row while the other 5 sit in the other row. The following information is known about them. 1. 2. 3. 4. 5. 6.

The rows are one behind another. There are only 4 girls in the group - Shalini, Priya, Priyanka and Sonia. Each row has an equal number of girls. Shalini does not sit in the same row as Ram. Priya and Priyanka sit in different rows. Ramesh and Suresh have to sit together if Karan and Sonia sit together and vice versa. 7. Karan sits in the 1st row. 8. If Abeer sits in the 1st row, then Prem sits behind Suresh otherwise Suresh sits behind Prem.

9. Shalini always sits to the left of Ramesh. If Ram sits in the 1st row, who can sit to his left from the following? 1) Suresh 2) Abeer 3) Ramesh 4) Prem 5) Cannot be determined Solution: If Ram is sitting in the 1st row, Shalini has to sit in the 2nd row. Shalini always sits to the left of Ramesh and hence Ramesh is also sitting in the 2nd row. 17. Now, one of Priya or Priyanka must sit in the 2nd row. 3 Marks Now, if Karan and Sonia sit separately, then Sonia has to sit in the 2nd row (as Karan always sits in the 1st row). However, this is not possible as there are already 2 girls in the 2nd row. Thus, Karan and Sonia sit together in 1st row which means Suresh has to sit with Ramesh in the 2nd row. Thus, Prem has to sit in the 1st row which means Abeer is in the 2nd row. Of the given options, only Prem is seated in the 1st row. Hence, option 4. If Sonia sits in the 2nd row, who will definitely be seated in the 1st row? 1) Ramesh 2) Suresh 3) Abeer 4) Priya 5) Cannot be determined Solution: Now, one of Priya or Priyanka would be seated with Sonia in the 2nd row. 18. ∴ Shalini has to sit in the 1st row which means Ramesh would also sit in the 3 1st row. Marks Also, Ram has to sit in the 2nd row. Since Karan always sits in the 1st row and Sonia is now in the 2nd row, they are sitting separately. ∴ Ramesh and Suresh should be sitting in different rows i.e. Suresh has to sit in the 2nd row. Now, if Abeer sits in the 1st row, Prem has to sit behind Suresh.

But, since Suresh is in the 2nd row, Prem has to sit in the 1st row which means Abeer is in the 2nd row. Hence, option 1. Boys and girls are alternately seated in the 1st row. If Sonia sits in the 1st row and Suresh is sitting at the right most end of one of the rows, who will occupy the 3rd place in the 1st row? 1) Prem 2) Abeer 3) Karan 4) Ram 5) Cannot be determined Solution: If Sonia sits in 1st row, she and Karan are sitting together. ∴ Ramesh and Suresh are also sitting together. Let us assume that Suresh is in the right most end of the 1st row. Now, one of Priya or Priyanka have to sit in the 1st row. Since all the five seats in the 1st row are occupied, Shalini will have to sit in 19. the 2nd row with Ram, which contradicts the given data. 3 Marks ∴ Suresh is sitting in the right most end of the 2nd row. Now, Prem must sit ahead of Suresh and hence he occupies the right most end of the 1st row. i.e. Abeer is in the 2nd row. Ramesh will sit in the 2nd row with Suresh and Shalini will be to the left of Ramesh and the remaining vacant spot will be occupied by one of Priya/Priyanka. ∴ People sitting in the 1st row: Karan, Sonia, Ram, Priya/Priyanka, Prem. We only know Prem's position and that girls and boys are sitting alternately. ∴ One of Karan or Ram can sit in the 3rd place. Hence, option 5. If “CHIKU” = 20 and “BRINJAL” = 42, what is the value of “PINEAPPLE”? 1) 50 20. 2) 72 3 3) 44 Marks 4) 69 5) 41

Solution: The logic here is that the number of letters in the word is multiplied by 1 less than the count. The word “CHIKU” has 5 letters. ∴ It is coded as 5 × 4 = 20. Similarly, the word “BRINJAL” has 7 letters, hence coded as 7 × 6 = 42. Using the same logic, “PINEAPPLE” is coded as 9 × 8 = 72. Hence, option 2. Four of the five options are equal in magnitude. Which of the options has a magnitude different from the others? 1. 5465 − 4287 2. 1178 + 2145 − 1015 − 1130 3. (1100 ÷ 2) + (1884 ÷ 3) 4. 9 × 99 + 6 × 66 – 3 × 33 5. 666 + 333 + 111 + 66 + 2 21. 3 Marks

1) 1 2) 2 3) 3 4) 4 5) 5 Solution: 1. 5465 − 4287 = 1178 2. 1178 + 2145 − 1015 − 1130 = 1178 + 2145 − 2145 = 1178 3. (1100 ÷ 2) + (1884 ÷ 3) = 550 + 628 = 1178 4. 9 × 99 + 6 × 66 − 3 × 33 = 891 + 396 − 99 = 1287 − 99 = 1188 5. 666 + 333 + 111 + 66 + 2 = 1178

Hence, option 4. Group Question Answer the following questions based on the information given below. Four friends Ajay, Bob, Chetan and Dan live in house numbers 1, 2, 3 and 4, though not necessarily in the same order. They own the following brand of cars : Skoda, Sumo, Santro and Swift among them. No two persons own the same brand of car. The following information is known about them. A. B. C. D. E.

Neither Ajay nor Dan live in an even numbered house. The person in house 2 drives a Skoda. The owner of the Swift is in house number 4. Ajay does not own a Sumo. Bob drives the Swift.

In what house number does Ajay live? 1) 1 2) 3 3) 1 or 3 4) 2 5) 2 or 4 Solution: There are 3 parameters here : Person, Brand of Car and House number. Now, Bob drives the Swift and the owner of the Swift stays in house number 4. Therefore, Bob lives in house number 4. ∴ Bob - Swift - House 4. Neither Ajay nor Dan stay in even numbered houses. 22. 3 Therefore, these two stay in house numbers 1 and 3 (in no specific order). Marks Hence, Chetan has to stay in house no 2. Since the person in house number 2 drives a Skoda, Chetan drives a Skoda. ∴ Chetan - Skoda - House 2. Now, Ajay can drive a Sumo or a Santro. However, it is given that Ajay does not drive a Sumo. Therefore, Ajay drives a Santro and Dan drives a Sumo. However, it cannot be determined whether Ajay stays in house number 1 or 3. He can stay in any one of the two. Hence, option 3. Which car does Dan drive? 1) Santro 2) Sumo 3) Swift 23. 4) Skoda 3 5) Cannot be determined Marks Solution: From the solution to the first question of this set, it is obvious that Dan drives the Sumo. Hence, option 2. 24. If Ajay lives in house 1, then in which house does Dan live? 3 1) 2 Marks 2) 3

3) 4 4) 3 or 4 5) None of these Solution: From the solution to the first question of this set, Ajay and Dan stay in house numbers 1 and 3 (in no specific order). Therefore, if Ajay stays in house number 1, Dan has to stay in house number 3. Hence, option 2. On a particular evening, Mansi was heading home from office, walking towards the north. She crossed her daughter Latika mid way on the route. Mansi observed that her shadow was falling towards Latika‟s right. In which direction was Latika moving? 1) North 2) South 3) East 4) West 5) Indeterminate Solution: In the evening, since the sun is in the west, the shadow of an object will fall towards the east. Since Mansi was walking towards the north, her shadow fell on her right hand side. For this shadow to fall on Latika's right, there can be two possiblities: 25. Case 1 : Latika was walking towards the north. 3 Marks If Latika was also walking towards the north and was either on the left of Mansi or parallel to Mansi (on the right side) such that the distance between Latika and Mansi (at the point of crossing) was less than the length of the shadow, Mansi's shadow would fall on Latika's right. Hence, the given condition would be satisfied. Case 2 : Latika was walking towards the south. If Latika was walking towards the south and was parallel to Mansi (on the right side) such that the distance between Latika and Mansi (at the point of crossing) was more than the length of the shadow, Mansi's shadow would fall on Latika's right. Hence, the given condition would still be satisfied. Hence, Latika could have been walking towards either the north or the south.

Hence, option 5. Group Question Answer the following questions based on the information given below. The columns and rows of matrix 1 are numbered 0 to 4 and that of matrix 2 from 5 to 9. A letter from these matrices is represented first by its row and next by column number e.g. O can be represented as 23, 34 and so on. In each of the questions that follow, find the set of codes that represents the word in that corresponding question.

MORAL 1) 43, 11, 77, 02, 96 2) 32, 33, 75, 44, 85 3) 22, 23, 66, 21, 65 4) 01, 23, 55, 03, 77 5) 14, 23, 55, 33, 69 Solution: M can be coded as 01, 14, 22, 32 and 43 O can be coded as 00, 11, 23, 34 and 40. 26. Hence, option 2 can be eliminated. 3 Marks R can be coded as 55, 66, 75, 88 and 99. Hence, option 1 can be eliminated. A can be coded as 02, 10, 21, 33 and 44. Hence, option 4 can be eliminated. L can be coded as 56, 69, 77, 85 and 96.

Hence, option 3 can be eliminated. Only option 5 contains all the correct codes. Hence, option 5. TONE 1) 31, 23, 59, 68 2) 04, 11, 76, 59 3) 20, 32, 67, 89 4) 31, 00, 87, 57 5) 31, 43, 59, 65 Solution: T can be coded as 04, 12, 20, 31 and 42. O can be coded as 00, 11, 23, 34 and 40. 27. 3 MarksHence, options 3 and 5 can be eliminated. N can be coded as 59, 67, 78, 86 and 95. Hence, options 2 and 4 can be eliminated. E can be coded as 57, 68, 76, 89 and 98. Only option 1 contains all the correct codes. Hence, option 1. KITE 1) 87, 31, 04, 76 2) 65, 13, 24, 89 3) 97, 24, 02, 57 4) 65, 03, 12, 98 5) 79, 30, 98, 12 Solution: K can be coded as 58, 65, 79, 87 and 97. 28. I can be coded as 03, 13, 24, 30 and 41. 3 Marks Hence, option 1 can be eliminated. T can be coded as 04, 12, 20, 31 and 42. Hence, options 2, 3 and 5 can be eliminated. Only option 4 contains all the correct codes. Hence, option 4. 29. 3

In a certain code language, if the word MAGIC is coded as 16-24-10-6-6, and the

Marks word BREAD is coded as 5-15-8-24-7, then how is the word RITUAL coded in that language? 1) 21-6-23-18-5-9 2) 21-6-25-18-4-9 3) 21-7-23-18-4-9 4) 22-6-23-18-4-9 5) 21-6-23-18-4-9 Solution: Consider the alphabetical position of each letter of the word MAGIC. Word:

M

A

Position: 13

G

1

7

I

C

9

3

Logic: +3 −3 +3 −3 +3 Code: 16 24 10

6

6

Similarly, consider the alphabetical position of each letter of the word BREAD Word:

B

Position: 2

R

E

A

D

18 5

1

4

Logic: +3 −3 +3 −3 +3 Code:

5 15

8

24

7

Thus, starting from the letter at the extreme left, each letter is alternately replaced by a letter 3 places after it (or before it) in the alphabet. Similarly, consider the position of each letter of the word RITUAL. Word:

R

I

Position: 18 9

T

U

A

20

21 1

L 12

Logic: +3 −3 +3 −3 +3 −3 Code:

21 6

23

18

4

9

So, the code for the word „RITUAL‟ is „21-6-23-18-4-9‟. Hence, option 5. If the representation (symbols) of Addition and Subtraction are interchanged and 30. representation (symbols) of Multiplication and Division are interchanged, then 3 what is the value of the following expression (L – M)/(X + M) – (M + I) × (L – Marks I)?

The values of the Variables are: I=1 L = 10 M = 100 V = 500 X = 1000 1) 11000 2) 99109 3) 110001 4) 99009 5) None of these Solution: The expression can be decoded as, (L – M)/(X + M) – (M + I) × (L – I) = (L + M) × (X – M) + (M – I)/(L + I) = (10 + 100) × (1000 – 100) + (100 – 1)/(10 + 1) = 99009 Hence, option 4. Read the two statements given below and choose the statement that logically follows from the first two. A. All laborers are workers. B. Some workers are builders. 31. 3 Marks

1) Some laborers are builders. 2) All laborers are builders. 3) Some builders are workers. 4) All laborers may be builders. 5) Some laborers are not builders. Solution:

A possibility arising from the two given statements can be represented in the form of a Venn diagram as shown. Options 1 and 2 are incorrect as the workers who are laborers may not be builders. Option 3 is incorrect as it logically follows only from statement B. Option 4 is correct because of the word „may‟ which means that at least in one of the cases, all laborers are builders. Option 5 is incorrect because there can be a case where all laborers are builders. Hence, option 4.

32. 3 Marks

Given below is a passage followed by several statements that can be drawn from the facts stated in the passage. Examine each statement separately in the context of the passage and decide whether they are implied from the passage. Not so long ago, Chongqing hardly seemed destined for greatness. Quite the contrary. It was by no means inevitable that a misty, ancient town built on the bluffs of the Yangtze would become the fastest-sprawling global metropolis. Indeed, the city‟s meteoric rise is difficult to explain by logic alone. But it is emblematic: Timing, luck, geography and government largesse all played a part. Statements: I. II. III.

Surprisingly enough, Chongqing is not a great city now. Chongqing became great very fast. Chongqing‟s government was not a factor in its success.

1) Only I is implicit.

2) Only II is implicit. 3) Both I and III are Implicit. 4) Both II and III are implicit. 5) Both I and II are implicit. Solution: Statement II is implied from „meteoric rise‟, „destined for greatness‟ and „fastestsprawling global metropolis‟ Statements I and III contradict the data given in the passage. Hence, the correct answer is option 2. The result of class 10 is declared. Half of those who failed in English managed to pass in Mathematics and 1/3rd of who failed in Mathematics managed to pass in English. Every student of this class passed in exactly one subject. The ratio of the number of students who passed in English to the number of students who passed in Mathematics is, 1) 1 : 3 2) 2 : 3 3) 1 : 2 4) 2 : 1 5) 3 : 2 Solution: Let x be the number of students that passed only in Mathematics, y be the number of students that passed only in English and z be the number of students that failed in both exams.

33. 3 Marks

Since every student of the class passed in exactly one subject, the number of students that passed in English = the number of students that passed in English only. The same logic applies for Mathematics. Then, the number of students that failed in Mathematics is y + z y + z = 3y ∴ z = 2y Similarly, The number of students who failed in English is x + z x + z = 2x ∴x=z ∴y:x=1:2

Hence, option 3. Group Question Answer the following questions based on the information given below.

Among 8 persons A, B, C, D, E, F, G and H, there are three married couples and two pairs of siblings. It is known that: i. ii. iii. iv.

D‟s father-in-law is E and F‟s daughter-in-law‟s brother is G. Members in each pair of siblings are of different genders. A‟s mother H likes B, the daughter of C and E. A is B‟s sister-in-law and two of the three married couples have two children each. Who all belong to the same generation? 1) F, A, G, E 2) F, H, C, G 3) A, H, B, C 4) A, D, C, G 5) A, D, B, G Solution: It is given that there are three couples, two pairs of siblings and two out of three couples have two children. If a rectangle represents males, a circle represents females and a double line represents a couple, the basic family tree would be:

34. 3 Marks From conditions (i) D‟s father-in-law is E and F‟s daughter-in-law‟s brother is G. (iii) B is the daughter of C and E. We know that D and B are the children of different couples. Also from (iv) and (i) A is a female and G is a male. ∴ A, D, G and B belong to the same generation. Also, as B is a female; B and G form one pair of siblings and A and D form the other. ∴ B and D form a married couple. We know C and E form a pair of married couple where E is a male. ∴ H and F are the other married couple.

We know that H is female, ∴ F is male. Thus the family tree obtained is:

A, D, B and G belong to the younger generation. Hence, option 5. Who are the males among the eight? 1) F, G, D, E 2) F, A, G, E 3) F, G, A, B 35. 4) E, D, G, A 3 Marks 5) G, C, D, B Solution: From the figure, we can see that F, G, D and E are males. Hence, option 1. How is D related to B? 1) Brother 2) Wife 3) Husband 36. 4) Sister 3 Marks 5) Father-in-law Solution: As it can be seen, D is B‟s husband. Hence, option 3. Group Question Answer the following questions based on the information given below. Shreya has with her a cube made up of 27 smaller cubes. She names each face of the larger cube with an alphabet from A–F. She then starts writing a number from 1–54 on each visible face of the smaller cube such that the face A gets the lowest sum followed by face B and so on. Shreya writes the digits in such a way that no two consecutive integers are written on a particular face. 37. What can be the least total on the face D? 3 1) 180

Marks

2) 162 3) 252 4) 168 5) None of these Solution: Face A < Face B < Face C < Face D Since there are 4 faces under consideration, consider the numbers to be in groups of 4 i.e. group size = number of faces under consideration. To make the inequality possible and to ensure the least total for face D, assign the first four natural numbers to faces A, B, C and D respectively. Now, if the same process is followed i.e. the number 5 is assigned to face A, the number on face D will be 8 i.e. the largest of this group of 4. This will not help minimise the sum. Also, if one assigns the largest number of a group of 4 numbers to the face where the smallest number was assigned in the previous iteration, the gap between the sum on the faces will not grow. Therefore, assign 5 to face B, 6 to face C, 7 to face D and 8 to face A. In general, follow the algorithm given below: 1) Assign the first four natural numbers to A, B, C and D respectively. 2) For the next four, assign the largest number of that group to the face that had the smallest number of the previous group. 3) Since each face will have 9 small cubes, follow this till 9n natural numbers are not exhausted. Here, n is the number of faces under consideration (in this case, 4). Thus, the numbers on the first four faces can be shown as follows:

Thus, the least possible total for the face D is 168. Hence, option 4. What is the least total on the face F? 1) 255 2) 258 3) 260 38. 4) 262 3 5) None of these Marks Solution: Using the logic given in the problem above, the numbers in the 6 cubes can be found as shown below.

Thus, the least total on face F is 255 Hence, option 1. What is the difference between the total on face A and face B when the total on face F is the least? 1) 1 2) 2 3) 3 4) 4 39. 5) 5 3 Solution: Marks Consider the arrangement of numbers obtained in the previous solution. In this arrangement, the total on face F is the least. In this case, the difference between the sum on face A and face B is 3.

40. 3

Hence, option 3. What is the maximum total for the face C? 1) 325

Marks

2) 327 3) 329 4) 331 5) None of these Solution: For the total on face C to be the highest, try and minimise the total on faces A and B. Each face has 9 numbers on it. To minimise the total on faces A and B, assign the first 18 natural numbers alternately to faces A and B respectively. Now, to maximise the total on face C, follow the same procedure as outlined in the first two questions staring from the number 19 and ending at 54. Hence, the final arrangement is as shown below.

As per this arrangement, the maximum total on face C is 327. Hence, option 2.

Section III Choose the word which is OPPOSITE in meaning to the word given in CAPITAL LETTERS: AZOIC 1) Animate 2) Sterile 3) Massive 4) Persistent 5) Azure 1. Solution: 3 Marks “Azoic” means „inanimate; without life‟. It is believed to be of an age that preceded the first appearance of life. It is characterised by having left no evidence of life in the form of organic remains. “Animate” means „living; possessing life‟. “Sterile” means „barren‟. “Massive” means „huge‟. “Persistent” means „determined to continue despite obstacles‟. “Azure” means „shades of Blue‟. Hence, the correct answer is option 1. Each of the questions below contains a paragraph followed by alternative summaries. Choose the option that best captures the essence of the paragraph. Cannibalism has been occasionally practiced as a last resort by people suffering from famine. In the US, the group of settlers known as the Donner party resorted to cannibalism while snowbound in the mountains for the winter. The last survivors of Sir John Franklin's Expedition were found to have resorted to cannibalism in their final push across King William Island towards the Back River. There are disputed claims that cannibalism was widespread during the famine of Ukraine in the 1930s, during the Siege of Leningrad in World War II, and during the Chinese Civil War and the Great Leap Forward in the People's Republic of China. There were also rumors of several cannibalism outbreaks during World War II in the Nazi concentration camps where the prisoners were malnourished. Cannibalism was also practiced by Japanese troops as recently as 2. World War II in the Pacific theater. A more recent example is of leaked stories 3 Marks from North Korean refugees of cannibalism practiced during and after a famine that occurred sometime between 1995 and 1997. History claims that Cannibalism, though uncommon, has been practiced for a very long time. Recent examples also show that cannibalism is not 1) restricted to a region or country but has been practiced, at some point in time, in various parts of the world. There were rumors of several cannibalism outbreaks during World War II in the Nazi concentration camps where the prisoners were malnourished. 2) Cannibalism was also practiced by Japanese troops as recently as World War II in the Pacific theater. The stories of cannibalism outbreak during World War II and Ukraine famine strengthen the claims of similar outbreaks elsewhere in history. 3) Some of the claims, though disputed, seem to be true based on such examples. Malnutrition, not religion, is believed to be the major reason for

such a practice. Cannibalism is a rare practice which has been followed in some parts of the world at times of crisis during wars and famines, which lead to people 4) taking such drastic measures. Traces of this practice have been found around the globe, in China, USA, Germany, Japan and many others. Cannibalism is only practiced at times of war and famines when there is a lack of food. This practice has been reported in several parts of the world. It 5) is uncommon, but is said to have broken out in USA, Germany, China, Japan and many others. Solution: The passage does not introduce or describe the practice of cannibalism. It only mentions it as being a last resort and goes on to give examples of the places where it has been reported to have broken out. Option 1, though close, does not cover all the points properly. It also does not mention any of the examples from the passage. Option 2 simply reproduces parts of the passage without summarizing it. Option 3 introduces external data- „religion‟. Option 5 states that it is „only‟ practiced at times of war and famines. The passage does not mention the exclusivity factor. Option 4 is correct as it covers all the points exactly the way they are mentioned in the passage without altering any piece of information. It also touches upon the examples briefly. Hence, the correct answer is option 4. Fill in the blanks in the passage with the most appropriate set of words from the options for each blank. Although the principle that events ____ the heavens are mirrored ____ those on Earth were once generally held ____ most traditions ____ astrology around the world, ____ the West there has historically been a debate among astrologers ____ the nature of the mechanism behind astrology. 1) in, by, in, of, in, over 3. 2) of, by, by, in, in, over 3 3) in, in, in, in, inside, over Marks 4) in, by, in, in, in, under 5) of, by, in, of, in, over Solution: Options 2 and 5 are incorrect as we need a preposition to describe the events that occur in the heavens. Hence „of‟ is an incorrect preposition for the first blank. Further, „in‟ signifies within the limits or boundary of. The correct preposition for the fourth blank will be „of‟. Options 3 and 4 can be eliminated. Hence, the correct answer is option 1. Group Question A passage is followed by questions pertaining to the passage. Read the passage and answer the questions. Choose the most appropriate answer. Chromosome linkage studies of sexual orientation have indicated the presence of multiple contributing genetic factors throughout the genome. In 1993, Dean Hamer and colleagues published findings from a linkage analysis of a sample of 76 gay brothers and their families. Hamer found that the gay men had more gay male uncles

and cousins on the maternal side of the family than on the paternal side. Gay brothers who showed this maternal pedigree were then tested for X chromosome linkage, using twenty-two markers on the X chromosome to test for similar alleles. In another finding, thirty-three of the forty sibling pairs tested were found to have similar alleles in the distal region of Xq28, which was significantly higher than the expected rates of 50% for fraternal brothers. This was popularly (but inaccurately) dubbed as the 'gay gene' in the media, causing significant controversy. A later analysis by Hu replicated and refined these findings. This study revealed that 67% of gay brothers in a new saturated sample shared a marker on the X chromosome at Xq28. Sanders replicated the study, finding 66% Xq28 marker sharing in 54 pairs of gay brothers. Although two other studies failed to find a preponderance of gay relatives in the maternal line of homosexual men, a rigorous replication of the maternal loading was reported on samples in Italy in England. One study by Rice failed to replicate the Xq28 linkage results. Meta-analysis of all available linkage data indicates a significant link to Xq28, but also indicates that additional genes must be present to account for the full heritability of sexual orientation. Mustanski performed a full-genome scan (instead of just an X chromosome scan) on individuals and families previously reported on in Hamer and Hu, as well as additional new subjects. With the larger sample set and complete genome scan, the study found somewhat reduced linkage for Xq28 than reported by Hamer et al. However, they did find other markers with significant likelihood scores at 8p12, 7q36 and 10q26. Interestingly, one of the links showed highly significant maternal loading, thus further confirming the previous family studies. A recent study suggests linkage between a mother's genetic make-up and homosexuality of her sons. Women have two X chromosomes, one of which is "switched off". The inactivation of the X chromosome occurs randomly throughout the embryo, resulting in cells that are mosaic with respect to which chromosome is active. In some cases though, it appears that this switching off can occur in a nonrandom fashion. Bocklandt reported that, in mothers of homosexual men, the number of women with extreme skewing of X chromosome inactivation is significantly higher than in mothers without gay sons. 13% of mothers with one gay son, and 23% of mothers with two gay sons showed extreme skewing, compared to 4% percent of mothers without gay sons. Blanchard and Klassen reported that each older brother increases the odds of a man being gay by 33%. This is now "one of the most reliable epidemiological variables ever identified in the study of sexual orientation." To explain this finding, it has been proposed that male fetuses provoke a maternal immune reaction that becomes stronger with each successive male fetus. Male fetuses produce HY antigens which are "almost certainly involved in the sexual differentiation of vertebrates." It is this antigen which maternal H-Y antibodies are proposed to both react to and 'remember'. Successive male fetuses are then attacked by H-Y antibodies which somehow decrease the ability of H-Y antigens to perform their usual function in brain masculinisation. 4. Which of these factors may not lead to a gay offspring? 3 1) A gay maternal uncle Marks 2) A marker on the X chromosome at Xq28

3) Extreme skewing of X chromosome activation 4) Three older brothers 5) Fewer H-Y antigens Solution: According to the first paragraph, a gay maternal uncle does increase the chances of a gay offspring. Therefore, option 1 is incorrect. According to the second paragraph, “This study revealed that 67% of gay brothers ... shared a marker on the X chromosome at Xq28.” Therefore, a marker on the X chromosome at Xq28 may be found in gay offspring, making option 2 incorrect. Option 4 is incorrect as the last paragraph states, “Blanchard and Klassen reported that each older brother increases the odds of a man being gay by 33%.” According to the last paragraph, “Male fetuses produce HY antigens which are "almost certainly involved in the sexual differentiation of vertebrates." ... Successive male fetuses are then attacked by H-Y antibodies which somehow decrease the ability of H-Y antigens to perform their usual function in brain masculinisation.” Thus fewer H-Y antigens will result in a decreased brain masculinisation function, making it more likely to lead to homosexuality. Therefore, option 5 is incorrect. According to the fourth paragraph, “Bocklandt reported that, in mothers of homosexual men, the number of women with extreme skewing of X chromosome inactivation is significantly higher than in mothers without gay sons.” Thus, it is the extreme skewing of X chromosome INACTIVATION, and not activation, that may lead to a gay offspring. Hence, the correct answer is option 3. “Interestingly, one of the links showed highly significant maternal loading, thus further confirming the previous family studies.” Why does the author begin this statement with the word “interestingly”? Because though the study‟s focus was to use a full genome scan to determine the veracity of the “gay gene” and to see if any other genes 1) affected homosexuality, it also ended up gathering evidence in support of the much debated topic of maternal loading. Because it is interesting that maternal loading is still debated when the 2) evidence clearly states otherwise; a full genome scan reveals that genetic factors are responsible for homosexuality. Genetic markers like the ones on 8p12, 7q36 and 10q26 prove that 5. 3) homosexuality is a genetic defect, as revealed by a full genome scan, 3 and that maternal loading has very little to do with it. Marks It is interesting to note that maternal loading was proven in a failed 4) study designed to determine how much genetic markers were related to homosexuality. The fact that the more the number of gay maternal uncles an offspring 5) has, the greater his chances of being a homosexual is interesting. Solution: By using the word “interestingly” the author is trying to draw the reader‟s attention to the fact that the connection to maternal loading was discovered even though the scientists were looking for something else. The only option that expresses this is 1. Option 2 is wrong as there is no real evidence disproving maternal loading.

Option 3 is wrong as the passage states that maternal loading is proven even in a full genome scan. Option 4 is wrong as it calls the genome scan study “failed” when clearly, it is a success. Option 5 is wrong as even though it is an interesting fact, that is not the reason why the author has used that word. Hence, the correct answer is option 1. Which among the following can be a result of reduced H-Y antigen production? A. Not enough masculinisation of the brain of the male foetus. B. Production of excessive feminine hormones in a male foetus. C. A high probability of homosexuality in the male foetus. D. An ability to fight H-Y antibodies. 1) A only 2) B only 3) A and C 4) A and D 6. 5) All of the above 3 Solution: Marks Statement A is correct because it is stated in the passage that “Successive male fetuses are then attacked by H-Y antibodies which somehow decrease the ability of H-Y antigens to perform their usual function in brain masculinisation.” Option C is correct as an inability to perform brain masculinisation can lead to a greater chance of homosexuality in the foetus. Option B is wrong as there is no evidence in the passage to prove it. Option D is wrong as the passage says nothing about the ability of a male foetus to fight H-Y antibodies. Hence, the correct answer is option 3. The word most likely to replace “preponderance” in the passage is: 1) Unbelievable 2) Thoughtfulness 3) Timeliness 4) Predominance 5) Minority 7. Solution: 3 The correct answer is 4. “Preponderance” means „superiority in numbers‟. Marks The word that comes closest in meaning is predominance. Unbelievable means incredible, therefore option 1 is incorrect. Preponderance is not related to thoughtfulness or timeliness, making options 2 and 3 incorrect. Option 5 is incorrect as „minority‟ is an antonym. Hence, the correct answer is option 4. The option that best summarizes this passage is: In 1993, Dean Hamer and his colleagues discovered maternal loading. 8. Hu built upon these findings, while Mustanski confirmed them. Other 1) 3 factors, such as the mother‟s genetic makeup and the number of siblings Marks also contributed. Genes play a large role in sexual orientation of a male foetus. Some 2) contributing factors include a dominance of male gay cousins and uncles

on the mother‟s side, a few genetic markers, and older brothers. Also, HY antigens are responsible for brain masculinization. Genetic factors have been proven to be responsible for homosexuality. The most repetitive finding has been that of maternal loading, which can 3) therefore be credited as a major factor. Besides it, there are others like genetic markers, skewing of X-chromosome in a mother and the presence of older brothers. A male foetus can be born homosexual if and only if there‟s a heavy 4) presence of gay men among his maternal uncles and cousins. Many genetic factors contribute towards sexual orientation, including maternal loading, markers on the X-Chromosome at Xq28, along with a 5) few other genes, extreme skewing of X-chromosome inactivation and having older brothers. Solution: Option 1 is incorrect since who conducted those experiments isn‟t as important as the results. Option 2 is wrong as it is vague about the main points and randomly talks about H-Y antigens. Option 3 is wrong as the passage never mentions anything about how maternal loading is a “major” factor. Option 4 is wrong as it incorrect mentions only one point. Option 5 succinctly paraphrases the key points of the passage, which are not the studies, but the results. It also mentions the many genetic factors that affect homosexuality, as mentioned in the passage. Hence, the correct answer is option 5. In the following, a sentence is fragmented into four parts. Choose for your answer the fragment that carries the error. Ignore punctuation errors if any. Despite his youth, (1) he is someone (2) who I respect (3) a lot.(4) 1) 1 2) 2 3) 3 9. 4) 4 3 5) No error Marks Solution: There is often an error in the usage of who and whom. The basic difference between the two pronouns is that “who” is used as the subject of the verb and “whom” is used as the object of the verb. In the given sentence, the subject is “I” and the highlighted word refers to the object of the verb “respect” – I respect him. Therefore, “whom” should be used. Hence, the correct answer is option 3. Each of the following questions has a sentence/paragraph with two underlined words that do not make sense. Choose the most appropriate replacement for the words from the options given. 10. In texts that are written according to this view, the gerontological alchemical 3 symbols, diagrams, and textual imagery of late alchemical works typically contain Marks multiple layers of meanings, allegories, and references to other equally cryptic works; and must be laboriously “decoded” in order to discover their positron meaning. 1) cryptic, inferred

2) cryptic, true 3) ambiguous, phaneritic 4) apocryphal, esoteric 5) perplexing, plain Solution: “Cryptic” means „puzzling or based on a code‟. “Ambiguous” is „vague, by accident or intent‟. “Apocryphal” means „of doubtful authorship or authenticity‟. “Perplexing” means „complicated or confusing‟. The phrase, “other equally cryptic works” in the latter part of the sentence gives us a clue that “cryptic” would be the correct choice for the first blank. However, since the other words for the first blank among the given options have similar meanings we will consider all the words to be suitable and look for our answer from the second blank. “Phaneritic” is „related to rocks that have visible crystals‟. It is completely unrelated to the context. This eliminates option 3. “Esoteric” means „understood only by a select few‟. Since the symbols, diagrams and textual imagery contains multiple layers of meanings , allegories and references to other equally cryptic works, they must be laboriously decoded in order to discover their real or true meaning. Thus the second blank cannot have a meaning that is „inferred‟, or understood by only a select few. Eliminate options 1 and 4. Moreover, the word “plain” implies that the meaning is already clear and straightforward - i.e. it would not have to be “decoded”. This eliminates option 5. Hence, the correct answer is option 2. Select the option which is having similar analogy vis-a-vis the analogy given in the question. Seductive: Tempting :: 1) Irascible : Irritable 2) Intense: Aloof 3) Shady: Righteous 11. 4) Vivid: Exalted 3 5) Shrewd: Rampant Marks Solution: “Seductive” is a synonym of “tempting”. Options 2, 3, 4 and 5 do not share the same relationship as the key pair. The words in option 3 can be termed as antonyms. Option 1 is the most logical choice as the two words in the option also share a synonymous relationship. Hence, the correct answer is option 1. In the following, a sentence is fragmented into four parts. Choose for your answer the fragment that carries the error. Ignore punctuation errors if any. Those who live in (1) reasonable happy surroundings (2) sustain positive 12. feelings (3) and remain happy.(4) 3 1) 1 Marks 2) 2 3) 3 4) 4 5) No Error

Solution: The adjective “reasonable” has been incorrectly used in place of the adverb “reasonably”. Hence, the correct answer is option 2. Answer the following questions based on the information given below. Which of the following words means that a person feels worried and is slightly afraid of something? 1) concerned 2) nervous 3) overwrought 13. 4) uneasy 3 5) destressed Marks Solution: „Concerned‟ means „feeling concern about somebody‟. „Nervous‟ is „to feel worried and slightly afraid‟. „Overwrought‟ means „worked up; excessively excited or agitated‟. „Uneasy‟ means „uncomfortable‟. „destressed‟ means „having no tension‟. Hence, the correct answer is option 2. Select the odd man out from the given alternatives. 1) Perspicacity 2) Acumen 3) Discernment 14. 4) Humour 3 5) Acuity Marks Solution: Perspicacity, acumen, discernment and acuity all mean the ability to understand and reason. The odd one out is humour because although it refers to a mental ability it is not directly related to understanding. Hence, the correct answer is option 4. Answer the following question based on the information given below. Which of the following is a synonym of 'storm'? 1) Hera 2) Squall 3) Torpedo 15. 4) Stroma 3 5) Croma Marks Solution: “Squall” is „a sudden, violent gust of wind, often accompanied by rain, snow, or sleet‟. The other words either do not exist or are unrelated to the meaning of storm. Hence, the correct answer is option 2. Group Question Fill in the blanks in the passage with the most appropriate set of words from the options for each blank.

Instructional Design is the practice of ___(1)___ the effectiveness, efficiency and appeal of instruction and other learning experiences. The process consists broadly of determining the current state and needs of the ___(2)___, defining the end ___(3)___of instruction, and creating some "intervention" to assist in the transition. The outcome of this instruction may be directly observable and ___(4)___ measured or completely hidden and ___(5)___. (1) 1) making 2) motivating 3) maximizing 4) contracting 5) contradicting 16. Solution: 3 The first blank requires a word that fits in the context of increasing the Marks effectiveness, efficiency and appeal of instruction. „Making‟ would require an object – making something more..... – which is missing. This eliminates option 1. Motivating, contracting and contradicting are contextually inappropriate. „Maximize‟ means „to increase to the greatest possible amount or degree‟ and is contextually appropriate. Hence, the correct answer is option 3. (2) 1) learner 2) people 3) designer 17. 4) populace 3 5) psychologist Marks Solution: The passage talks about instructional design as a learning experience. Therefore, the process has to determine the state and needs of the „learner‟. The other options do not fit this context. Hence, the correct answer is option 1. (3) 1) destination 2) design 3) motivation 4) cognition 18. 5) goal 3 Solution: Marks The passage talks about defining the end ___ of instruction. Therefore, the blank requires a word that means purpose or target or objective. „Goal‟ means „the aim or object towards which an endeavour is directed‟, and fits this context. The other options are contextually inappropriate. Hence, the correct answer is option 5. (4) 1) directly 19. 2) scientifically 3 3) physical Marks 4) accurate 5) exact

Solution: The passage talks about the outcome being directly observable and measured. The blank requires a word that describes the way the outcome is measured. For an outcome to be considered valid, its measurement must be „scientific‟, meaning it must be „systematic or accurate in the manner of an exact science‟. „Directly‟ in option 1 is an unnecessary repetition. Since the blank describes the verb „measured‟, the answer has to be an adverb. Options 3, 4 and 5 are adjectives, and are therefore grammatically incorrect. Hence, the correct answer is option 2. (5) 1) visible 2) conspicuous 3) distinct 4) assumed 5) apparent Solution: The last blank is in the context of an outcome being completely hidden. 20. According to the passage, “The outcome of this instruction may be directly 3 observable and ... measured OR completely hidden and ___.” The „or‟ in the Marks statement indicates that the blank has to contain a word that is the opposite of something that is directly observed and measured, and it has to be in consonance with being hidden. „Assumed‟, meaning „supposed or taken for granted‟ is the correct contrast to something that is directly observed and measured, and therefore hidden. The remaining words are antonyms of „assumed‟ and are therefore contextually incorrect. Hence, the correct answer is option 4. In each of the following sentences, part or the entire sentence is underlined. The answer-choices offer four ways of phrasing the underlined part. If you think the original sentence is better than the alternatives, choose 1 which merely repeats the underlined part; otherwise choose one of the alternatives. Written in the year 1971, N.S. Paul‟s „On the warfront‟ was the cause of a sensational nationwide controversy and he had to remain underground for two years. N.S. Paul‟s „On the warfront‟ was the cause of a sensational nationwide 1) controversy and N.S. Paul and his book, „On the warfront‟ were the causes of a sensational 21. 2) nationwide controversy and 3 Marks 3) N.S. Paul‟s „On the warfront‟ was the cause of a such a controversial nationwide sensation that „On the warfront‟ written by N.S. Paul was the cause of a nationwide 4) controversy so sensational that N.S. Paul‟s „On the warfront‟ was the cause of a nationwide controversy so 5) sensational that Solution: Since the sentence begins with a modifier, the noun that the modifier qualifies should be the subject of the sentence. In this case, it is the book „On the warfront‟ and not the author. Option 2 gets eliminated. Option 4 repeats the verb „written‟, hence can be eliminated.

Option 3 changes the sentence construction by exchanging the words „controversy‟ and „sensational‟. Between options 1 and 5, option 5 is a better fit as it correctly uses the construction „so…that‟ and it is more fitting than option 1. Hence, the correct answer is option 5. In each of the following sentences, part or the entire sentence is underlined. The answer-choices offer four ways of phrasing the underlined part. If you think the original sentence is better than the alternatives, choose 1 which merely repeats the underlined part; otherwise choose one of the alternatives. On Indian pitches, batsmen play spin with far more ease than most European pitches. 1) than most European pitches. 2) than on most European pitches. 22. 3) than they do on most European pitches. 3 4) than they play on most European pitches. Marks 5) than European pitches mostly. Solution: The correct option should have a grammatically correct comparison between the two actions separated by „than‟. The first half of the sentence has an action of „playing‟ which is being compared. This action is not represented in options 1, 2 and 5. Hence, these can be eliminated. Between options 3 and 4, option 3 is better as it does not repeat the verb „play‟ and replaces it with „do‟, hence making the sentence more refined. Hence, the correct answer is option 3. Group Question A passage is followed by questions pertaining to the passage. Read the passage and answer the questions. Choose the most appropriate answer. In psychiatry, the term neologism is used to describe the use of words that only have meaning to the person who uses them, independent of their common meaning. This is considered normal in children, but a symptom of thought disorder (indicative of a psychotic mental illness, such as schizophrenia) in adults. Use of neologisms may also be related to aphasia acquired after brain damage resulting from a stroke or head injury. Neologisms are often created by combining existing words (compound noun or adjective) or by giving words new and unique suffixes or prefixes. Portmanteaux are combined words that begin to be used commonly. Neologisms also can be created through abbreviation or acronym, by intentionally rhyming with existing words or simply through playing with sounds. Neologisms often become popular through memetics, by way of mass media, the Internet, and word of mouth, including academic discourse in many fields renowned for their use of distinctive jargon, and often become accepted parts of the language. Other times, however, they disappear from common use just as readily as they appeared. Whether a neologism continues as part of the language depends on many factors, probably the most important of which is acceptance by the public. It is unusual, however, for a word to enter common use if it does not resemble another

word or words in an identifiable way. When a word or phrase is no longer "new", it is no longer a neologism. Neologisms may take decades to become "old", however. Opinions differ on exactly how old a word must be to cease being considered a neologism. Which of the following statements, based on the passage, is most likely to be true? A neologist is one who proposes either a new doctrine or a new 1) interpretation of source material such as religious texts. 2) Neologisms might lose their novelty when more people start using them. Aphasia is the reason why many words from the English language have 3) now been forgotten. Academic fields with their own distinctive jargon particularly look 4) forward to neologisms. 5) Both 1 and 2 23. Solution: 3 Option 1 is ruled out since this passage is not related to the other meanings or Marks allusions to the term „neologism‟. This passage is concerned only with its reference to vocabulary. Option 3 is also ruled out as this is an exaggerated sentence that has no basis in the passage. Option 4 is also ruled out as this can also not be inferred from the information in the passage. Option 2 is mentioned in the passage: „When a word or phrase is no longer "new", it is no longer a neologism. Neologisms may take decades to become "old"‟. Hence, the correct answer is option 2. Choose the word in the passage that means „a word formed by blending sounds from two or more distinct words and combining their meanings‟. 1) Memetics 2) Acronym 3) Portmanteaux 24. 4) Jargon 3 5) Suffix Marks Solution: Portmanteaux refers to „a word formed by blending sounds from two or more distinct words and combining their meanings‟. For eg: „brunch‟, from „breakfast‟ and „lunch‟, or “Spanglish”, referring to speaking a mix of both Spanish and English spoken by bilingual people. Hence, the correct answer is option 3. Which of the following is not untrue in the context of the passage? I. II. 25. 3 Marks

Words can have more than one meaning. Adding a suffix or prefix to a word is called neologism.

1) Only I 2) Only II 3) Neither I nor II 4) Both I and II 5) Either I or II Solution:

Option 1 is correct because it is mentioned in the passage: “…the use of words that only have meaning to the person who uses them, independent of their common meaning”. Option 2 is incorrect because it isn‟t the adding of an existing suffix or prefix that makes a neologism, it is the adding of anew or unique suffix or prefix that leads to a neologism. Hence, the correct answer is option 1. Mark the correct option, which puts the parts of the sentence in right order: A. Infosys sent an official communique B. Following the bank's decision C. To be with Deutsche Network Services D. To the 150 employees giving them E. The option to leave the company F. For much higher salaries 1) ABCDEF 2) BADECF 3) EFCDBA 26. 4) ABDFEC 3 Marks 5) CFDEBA Solution: On reading the options one can clearly decipher the sentence to mean that 150 employees of Infosys have been offered much higher salaries by Deutsche Network Services. Keeping this in mind the ADECF link is clear. In A, Infosys sends a communique to the 150 employees mentioned in D. The communique gives these 150 employees the option to leave the companymentioned in E- and work for Deutsche Network (mentioned in C) for much higher salaries mentioned in F. Statement B can only be the introductory statement since it cannot be accomodated anywhere else. Hence, the correct answer is option 2. Mark the correct option, which puts the parts of the sentence in right order: A. To connect to it B. The more it made sense for other networks C. Over the past 15 years and the bigger it got D. As it has grown at an astonishing rate E. A network of networks F. Internet has become 27. 3 Marks

1) FEDCBA 2) ABCEFD 3) CABDFE 4) EBADFC 5) AFEDCB Solution: F is obviously the introductory statement since it introduces the topic being discussed in the paragraph- which is the internet. We can eliminate options 2,3,4 and 5. FE is a pair since “has become” in F can only combine with “a network” in order

to frame a grammatically correct sentence. DC is a pair since D mentions the internet's astonishing growth and C mentiones the timeframe in which it has taken place- 15 years. BA is also a pair. “For other networks” in B can only connect grammatically and logically with “to connect to it” mentioned in A. Hence, the correct answer is option 1. In the following, a sentence is fragmented into four parts. Choose for your answer the fragment that carries the error. Ignore punctuation errors if any. It was challenging (1) to study while working, (2) but she was resolved (3) to take up the challenge.(4) 1) 1 28. 2) 2 3 3) 3 Marks 4) 4 5) No error Solution: The sentence is grammatically correct. Hence, the correct answer is option 5. Fill in the blanks in the passage with the most appropriate set of words from the options for each blank. If we watched the lobster _____ its native element, we should see it climbing actively the submerged rocks, _____ which it delights to live, by means of its strong legs; or swimming by powerful strokes of its great tail, the appendages _____ whose sixth joint are spread out _____ a broad fan-like propeller: seize it, and it will show you that its great claws are no mean weapons of offence; suspend a piece of carrion _____ its haunts, and it will greedily devour it, tearing and crushing the flesh _____ means of its multitudinous jaws. 1) in, among, of, into, among, by 29. 2) from, through, of, into, among, by 3 3) in, among, from, in, among, by Marks 4) from, among, of, into, by, by 5) in, among, of, into, among, with Solution: One studies a species „in‟ its native element. Therefore, options 2 and 4 can be eliminated. Option 3 is incorrect as the phrase described- the appendages “of whose sixth joint are spread out”. The preposition „from‟ would be incorrect for this blank. The use of „by‟ is correct for the last blank- „...crushing the flesh by means of ...‟ This eliminates option 5. Option 1 uses the appropriate prepositions. Hence, the correct answer is option 1. This question consists of two capitalized words that have a certain relationship to each other, followed by a certain pair of words. Choose the pair that is 30. RELATED to each other in the same way as the capitalized pair. 3 SEX : GENDER :: Marks 1) Boy : Girl 2) Child : Children 3) Read : Literacy

4) Population : History 5) Genus : Genera Solution: “Sex” (male/female) determines the “gender” (masculine/feminine) of a person. „Literacy‟ is determined by „ability to read‟ „Children‟ is the plural of „Child‟. „Boy‟ and „girl‟ merely show different sexes. „History‟ is not determined by „population‟. „Genera‟ is the plural of „genus‟. Hence, the correct answer is option 3. In the following, a sentence is fragmented into four parts. Choose for your answer the fragment that carries the error. Ignore punctuation errors if any. Economic reforms brought foreign competition,(1) led to the privatisation of certain public sector industries, (2) open up sectors hitherto reserved for the public sector (3) and led to an expansion in the production of fast-moving consumer goods.(4) 1) 1 31. 2) 2 3) 3 4) 4 5) No error Solution: The verbs in fragments 1, 2 and 4 are in the past tense. Therefore, even fragment 3 should be in the past tense with “opened”. Hence, the correct answer is option 3. In the following, a sentence is fragmented into four parts. Choose for your answer the fragment that carries the error. Ignore punctuation errors if any. Trekking in the monsoons (1) sounds like a lot of fun (2), but look after (3) landslides and mudflows.(4) 1) 1 2) 2 3) 3 4) 4 32. 5) No error Solution: The verb phrase “look after” means „to take care of‟ and does not fit the context of the given sentence. The use of the word “but” in the given sentence indicates a contrary idea- “it can be fun, but ...” has to convey something that is NOT fun, something dangerous. “Look out for” means „watch out for; be concerned about‟. Look out for landslides and mudflows fits the context. Hence, the correct answer is option 3.

View more...

Comments

Copyright ©2017 KUPDF Inc.
SUPPORT KUPDF